Finitely generated projective modules are locally free












14












$begingroup$


Let $A$ be a commutative noetherian ring, and let $M$ be a finitely generated projective $A$-module. It is well known and easy to prove that $A$ is locally free in the sense that for every $p inoperatorname{Spec} A$, the module $M_p$ is a free $A_p$-module.



Is it true that projectives are also locally free in the following (more geometric?) sense:




There are elements $f_1,dots,f_n in A$ such that $(f_1,dots,f_n) = 1$, and such that $M_{f_i}$ is a free $A_{f_i}$-module for all $1le i le n$.




Is this true? if so, can you provide a reference or explain how to prove it?



Thanks!










share|cite|improve this question











$endgroup$

















    14












    $begingroup$


    Let $A$ be a commutative noetherian ring, and let $M$ be a finitely generated projective $A$-module. It is well known and easy to prove that $A$ is locally free in the sense that for every $p inoperatorname{Spec} A$, the module $M_p$ is a free $A_p$-module.



    Is it true that projectives are also locally free in the following (more geometric?) sense:




    There are elements $f_1,dots,f_n in A$ such that $(f_1,dots,f_n) = 1$, and such that $M_{f_i}$ is a free $A_{f_i}$-module for all $1le i le n$.




    Is this true? if so, can you provide a reference or explain how to prove it?



    Thanks!










    share|cite|improve this question











    $endgroup$















      14












      14








      14


      10



      $begingroup$


      Let $A$ be a commutative noetherian ring, and let $M$ be a finitely generated projective $A$-module. It is well known and easy to prove that $A$ is locally free in the sense that for every $p inoperatorname{Spec} A$, the module $M_p$ is a free $A_p$-module.



      Is it true that projectives are also locally free in the following (more geometric?) sense:




      There are elements $f_1,dots,f_n in A$ such that $(f_1,dots,f_n) = 1$, and such that $M_{f_i}$ is a free $A_{f_i}$-module for all $1le i le n$.




      Is this true? if so, can you provide a reference or explain how to prove it?



      Thanks!










      share|cite|improve this question











      $endgroup$




      Let $A$ be a commutative noetherian ring, and let $M$ be a finitely generated projective $A$-module. It is well known and easy to prove that $A$ is locally free in the sense that for every $p inoperatorname{Spec} A$, the module $M_p$ is a free $A_p$-module.



      Is it true that projectives are also locally free in the following (more geometric?) sense:




      There are elements $f_1,dots,f_n in A$ such that $(f_1,dots,f_n) = 1$, and such that $M_{f_i}$ is a free $A_{f_i}$-module for all $1le i le n$.




      Is this true? if so, can you provide a reference or explain how to prove it?



      Thanks!







      commutative-algebra projective-module






      share|cite|improve this question















      share|cite|improve this question













      share|cite|improve this question




      share|cite|improve this question








      edited Mar 20 '15 at 16:06









      user26857

      39.3k124183




      39.3k124183










      asked Jan 8 '11 at 16:02









      the Lthe L

      2,52812150




      2,52812150






















          3 Answers
          3






          active

          oldest

          votes


















          11












          $begingroup$

          Yes, this is true. See this Math Overflow question for a precise statement and a reference to its proof in Bourbaki's Commutative Algebra.



          This result is also stated in my commutative algebra notes, but the proof is not unfortunately not yet written up there. I certainly hope that this will be remedied soon though, as I will be teaching a course out of these notes starting on Monday. When the proof gets written, I will update this answer with a page number.



          Added: Here is something in the MO answer that I decided was worth a comment here. For finitely generated modules, this stronger version of local freeness is actually equivalent to projectivity, whereas the weaker "pointwise local freeness" is subtly weaker in general.






          share|cite|improve this answer











          $endgroup$









          • 4




            $begingroup$
            Thank you for the reference. For the intrested reader, I should mention that Bourbaki proves the following much stronger result: If $M$ is a finitely presented module and if for some prime $p$, the module $M_p$ is free, then there exists a neighborhood $D(f)$ of $p$ such that $M_f$ is free over $A_f$, so that "being free at a point" implies "being free at a neigborhood of a point".
            $endgroup$
            – the L
            Jan 8 '11 at 16:51






          • 4




            $begingroup$
            @anonymous: Dear anonymous, more generally, if $M,N$ are finitely presented modules and $M_p, N_p$ are isomorphic, then $M_f simeq N_f$ for some $f in A-p$. This is explained in more detail (and generality) in EGA IV-8, where it is proved that (in a certain sense) if ${R_alpha}$ is an inductive system of rings, then the category of f.p. modules over $varinjlim R_alpha$ is the "colimit of the categories of modules over $R_alpha$," which has many useful applications in reducing questions about arbitrary f.p. modules to questions about finitely generated modules over noetherian rings.
            $endgroup$
            – Akhil Mathew
            Jan 15 '11 at 18:40






          • 2




            $begingroup$
            (contd.) For instance, Grothendieck's generic flatness lemma (if $A$ noetherian integral domain, $B$ finite $A$-algebra, $M$ finite $B$-module, then there is $f in A$ such that $M_f$ is flat over $A$) works for finitely presented modules because any such module must "descend" to a finitely presented module over a noetherian (e.g. finitely generated) subring.
            $endgroup$
            – Akhil Mathew
            Jan 15 '11 at 18:41



















          4












          $begingroup$

          For future reference, I have written up a constructive and reasonably self-contained, if somewhat dense, proof (one page). The basic idea is to first verify that idempotent matrices over local rings are equivalent to diagonal matrices with entries $1$ and $0$, thus showing that finitely generated projective modules over local rings are free.






          share|cite|improve this answer











          $endgroup$













          • $begingroup$
            Well, I think there are simpler proofs for finitely generated projective modules over local rings are free. Never mind, +1.
            $endgroup$
            – user26857
            May 8 '15 at 16:40












          • $begingroup$
            @user26857: I would be interested to hear them! Are they constructive in the technical sense that they avoid the use of the law of excluded middle? I only know of my write-up and the one in Mulvey's gem Intuitionistic algebra and representations of rings.
            $endgroup$
            – Ingo Blechschmidt
            May 8 '15 at 16:54










          • $begingroup$
            math.stackexchange.com/questions/147754/…
            $endgroup$
            – user26857
            May 8 '15 at 17:15










          • $begingroup$
            @user26857: Thanks! The problem is that a proof beginning with "Consider a minimal system of generators" is not constructive, since constructively one cannot show that any inhabited subset of the natural numbers contains a minimal element. In some cases though, the problem can be avoided by a clever rewording; I didn't try that in this case hard enough yet.
            $endgroup$
            – Ingo Blechschmidt
            May 8 '15 at 18:03



















          0












          $begingroup$

          This answer shows that if its stalk $M_mathfrak{p}$ at $mathfrak{p}$ is free, then there is an open neighbourhood $mathfrak{p}in D(f)$ on which its value $M_f$ is also free.



          Proof: Reduce to the case that the natural map $Mto M_mathfrak{p}$ is an injection by localising some $f_0in A$. Now use the basis of $M_mathfrak{p}$ to give a surjection $alpha:A^nto M$ whose localisation at $mathfrak{p}$ is the isomorphism $A^n_mathfrak{p}to M_mathfrak{p}$. Thus $ker alpha_mathfrak{p}=0$, so $ker alpha_f=0$ for some $fin A$. Thus $M_fsimeq A^n_f$ is free, completing the proof.



          Thus the quasicompact space $text{Spec}A$ admits a finite cover by $D(f_i)$'s, on each of which $M_{f_i}$ is free. $text{Spec}A=cup D(f_i)$ means precisely that $(f_1,...,f_n)=A$.






          share|cite|improve this answer











          $endgroup$













            Your Answer





            StackExchange.ifUsing("editor", function () {
            return StackExchange.using("mathjaxEditing", function () {
            StackExchange.MarkdownEditor.creationCallbacks.add(function (editor, postfix) {
            StackExchange.mathjaxEditing.prepareWmdForMathJax(editor, postfix, [["$", "$"], ["\\(","\\)"]]);
            });
            });
            }, "mathjax-editing");

            StackExchange.ready(function() {
            var channelOptions = {
            tags: "".split(" "),
            id: "69"
            };
            initTagRenderer("".split(" "), "".split(" "), channelOptions);

            StackExchange.using("externalEditor", function() {
            // Have to fire editor after snippets, if snippets enabled
            if (StackExchange.settings.snippets.snippetsEnabled) {
            StackExchange.using("snippets", function() {
            createEditor();
            });
            }
            else {
            createEditor();
            }
            });

            function createEditor() {
            StackExchange.prepareEditor({
            heartbeatType: 'answer',
            autoActivateHeartbeat: false,
            convertImagesToLinks: true,
            noModals: true,
            showLowRepImageUploadWarning: true,
            reputationToPostImages: 10,
            bindNavPrevention: true,
            postfix: "",
            imageUploader: {
            brandingHtml: "Powered by u003ca class="icon-imgur-white" href="https://imgur.com/"u003eu003c/au003e",
            contentPolicyHtml: "User contributions licensed under u003ca href="https://creativecommons.org/licenses/by-sa/3.0/"u003ecc by-sa 3.0 with attribution requiredu003c/au003e u003ca href="https://stackoverflow.com/legal/content-policy"u003e(content policy)u003c/au003e",
            allowUrls: true
            },
            noCode: true, onDemand: true,
            discardSelector: ".discard-answer"
            ,immediatelyShowMarkdownHelp:true
            });


            }
            });














            draft saved

            draft discarded


















            StackExchange.ready(
            function () {
            StackExchange.openid.initPostLogin('.new-post-login', 'https%3a%2f%2fmath.stackexchange.com%2fquestions%2f16814%2ffinitely-generated-projective-modules-are-locally-free%23new-answer', 'question_page');
            }
            );

            Post as a guest















            Required, but never shown

























            3 Answers
            3






            active

            oldest

            votes








            3 Answers
            3






            active

            oldest

            votes









            active

            oldest

            votes






            active

            oldest

            votes









            11












            $begingroup$

            Yes, this is true. See this Math Overflow question for a precise statement and a reference to its proof in Bourbaki's Commutative Algebra.



            This result is also stated in my commutative algebra notes, but the proof is not unfortunately not yet written up there. I certainly hope that this will be remedied soon though, as I will be teaching a course out of these notes starting on Monday. When the proof gets written, I will update this answer with a page number.



            Added: Here is something in the MO answer that I decided was worth a comment here. For finitely generated modules, this stronger version of local freeness is actually equivalent to projectivity, whereas the weaker "pointwise local freeness" is subtly weaker in general.






            share|cite|improve this answer











            $endgroup$









            • 4




              $begingroup$
              Thank you for the reference. For the intrested reader, I should mention that Bourbaki proves the following much stronger result: If $M$ is a finitely presented module and if for some prime $p$, the module $M_p$ is free, then there exists a neighborhood $D(f)$ of $p$ such that $M_f$ is free over $A_f$, so that "being free at a point" implies "being free at a neigborhood of a point".
              $endgroup$
              – the L
              Jan 8 '11 at 16:51






            • 4




              $begingroup$
              @anonymous: Dear anonymous, more generally, if $M,N$ are finitely presented modules and $M_p, N_p$ are isomorphic, then $M_f simeq N_f$ for some $f in A-p$. This is explained in more detail (and generality) in EGA IV-8, where it is proved that (in a certain sense) if ${R_alpha}$ is an inductive system of rings, then the category of f.p. modules over $varinjlim R_alpha$ is the "colimit of the categories of modules over $R_alpha$," which has many useful applications in reducing questions about arbitrary f.p. modules to questions about finitely generated modules over noetherian rings.
              $endgroup$
              – Akhil Mathew
              Jan 15 '11 at 18:40






            • 2




              $begingroup$
              (contd.) For instance, Grothendieck's generic flatness lemma (if $A$ noetherian integral domain, $B$ finite $A$-algebra, $M$ finite $B$-module, then there is $f in A$ such that $M_f$ is flat over $A$) works for finitely presented modules because any such module must "descend" to a finitely presented module over a noetherian (e.g. finitely generated) subring.
              $endgroup$
              – Akhil Mathew
              Jan 15 '11 at 18:41
















            11












            $begingroup$

            Yes, this is true. See this Math Overflow question for a precise statement and a reference to its proof in Bourbaki's Commutative Algebra.



            This result is also stated in my commutative algebra notes, but the proof is not unfortunately not yet written up there. I certainly hope that this will be remedied soon though, as I will be teaching a course out of these notes starting on Monday. When the proof gets written, I will update this answer with a page number.



            Added: Here is something in the MO answer that I decided was worth a comment here. For finitely generated modules, this stronger version of local freeness is actually equivalent to projectivity, whereas the weaker "pointwise local freeness" is subtly weaker in general.






            share|cite|improve this answer











            $endgroup$









            • 4




              $begingroup$
              Thank you for the reference. For the intrested reader, I should mention that Bourbaki proves the following much stronger result: If $M$ is a finitely presented module and if for some prime $p$, the module $M_p$ is free, then there exists a neighborhood $D(f)$ of $p$ such that $M_f$ is free over $A_f$, so that "being free at a point" implies "being free at a neigborhood of a point".
              $endgroup$
              – the L
              Jan 8 '11 at 16:51






            • 4




              $begingroup$
              @anonymous: Dear anonymous, more generally, if $M,N$ are finitely presented modules and $M_p, N_p$ are isomorphic, then $M_f simeq N_f$ for some $f in A-p$. This is explained in more detail (and generality) in EGA IV-8, where it is proved that (in a certain sense) if ${R_alpha}$ is an inductive system of rings, then the category of f.p. modules over $varinjlim R_alpha$ is the "colimit of the categories of modules over $R_alpha$," which has many useful applications in reducing questions about arbitrary f.p. modules to questions about finitely generated modules over noetherian rings.
              $endgroup$
              – Akhil Mathew
              Jan 15 '11 at 18:40






            • 2




              $begingroup$
              (contd.) For instance, Grothendieck's generic flatness lemma (if $A$ noetherian integral domain, $B$ finite $A$-algebra, $M$ finite $B$-module, then there is $f in A$ such that $M_f$ is flat over $A$) works for finitely presented modules because any such module must "descend" to a finitely presented module over a noetherian (e.g. finitely generated) subring.
              $endgroup$
              – Akhil Mathew
              Jan 15 '11 at 18:41














            11












            11








            11





            $begingroup$

            Yes, this is true. See this Math Overflow question for a precise statement and a reference to its proof in Bourbaki's Commutative Algebra.



            This result is also stated in my commutative algebra notes, but the proof is not unfortunately not yet written up there. I certainly hope that this will be remedied soon though, as I will be teaching a course out of these notes starting on Monday. When the proof gets written, I will update this answer with a page number.



            Added: Here is something in the MO answer that I decided was worth a comment here. For finitely generated modules, this stronger version of local freeness is actually equivalent to projectivity, whereas the weaker "pointwise local freeness" is subtly weaker in general.






            share|cite|improve this answer











            $endgroup$



            Yes, this is true. See this Math Overflow question for a precise statement and a reference to its proof in Bourbaki's Commutative Algebra.



            This result is also stated in my commutative algebra notes, but the proof is not unfortunately not yet written up there. I certainly hope that this will be remedied soon though, as I will be teaching a course out of these notes starting on Monday. When the proof gets written, I will update this answer with a page number.



            Added: Here is something in the MO answer that I decided was worth a comment here. For finitely generated modules, this stronger version of local freeness is actually equivalent to projectivity, whereas the weaker "pointwise local freeness" is subtly weaker in general.







            share|cite|improve this answer














            share|cite|improve this answer



            share|cite|improve this answer








            edited Apr 13 '17 at 12:58









            Community

            1




            1










            answered Jan 8 '11 at 16:32









            Pete L. ClarkPete L. Clark

            80.6k9161312




            80.6k9161312








            • 4




              $begingroup$
              Thank you for the reference. For the intrested reader, I should mention that Bourbaki proves the following much stronger result: If $M$ is a finitely presented module and if for some prime $p$, the module $M_p$ is free, then there exists a neighborhood $D(f)$ of $p$ such that $M_f$ is free over $A_f$, so that "being free at a point" implies "being free at a neigborhood of a point".
              $endgroup$
              – the L
              Jan 8 '11 at 16:51






            • 4




              $begingroup$
              @anonymous: Dear anonymous, more generally, if $M,N$ are finitely presented modules and $M_p, N_p$ are isomorphic, then $M_f simeq N_f$ for some $f in A-p$. This is explained in more detail (and generality) in EGA IV-8, where it is proved that (in a certain sense) if ${R_alpha}$ is an inductive system of rings, then the category of f.p. modules over $varinjlim R_alpha$ is the "colimit of the categories of modules over $R_alpha$," which has many useful applications in reducing questions about arbitrary f.p. modules to questions about finitely generated modules over noetherian rings.
              $endgroup$
              – Akhil Mathew
              Jan 15 '11 at 18:40






            • 2




              $begingroup$
              (contd.) For instance, Grothendieck's generic flatness lemma (if $A$ noetherian integral domain, $B$ finite $A$-algebra, $M$ finite $B$-module, then there is $f in A$ such that $M_f$ is flat over $A$) works for finitely presented modules because any such module must "descend" to a finitely presented module over a noetherian (e.g. finitely generated) subring.
              $endgroup$
              – Akhil Mathew
              Jan 15 '11 at 18:41














            • 4




              $begingroup$
              Thank you for the reference. For the intrested reader, I should mention that Bourbaki proves the following much stronger result: If $M$ is a finitely presented module and if for some prime $p$, the module $M_p$ is free, then there exists a neighborhood $D(f)$ of $p$ such that $M_f$ is free over $A_f$, so that "being free at a point" implies "being free at a neigborhood of a point".
              $endgroup$
              – the L
              Jan 8 '11 at 16:51






            • 4




              $begingroup$
              @anonymous: Dear anonymous, more generally, if $M,N$ are finitely presented modules and $M_p, N_p$ are isomorphic, then $M_f simeq N_f$ for some $f in A-p$. This is explained in more detail (and generality) in EGA IV-8, where it is proved that (in a certain sense) if ${R_alpha}$ is an inductive system of rings, then the category of f.p. modules over $varinjlim R_alpha$ is the "colimit of the categories of modules over $R_alpha$," which has many useful applications in reducing questions about arbitrary f.p. modules to questions about finitely generated modules over noetherian rings.
              $endgroup$
              – Akhil Mathew
              Jan 15 '11 at 18:40






            • 2




              $begingroup$
              (contd.) For instance, Grothendieck's generic flatness lemma (if $A$ noetherian integral domain, $B$ finite $A$-algebra, $M$ finite $B$-module, then there is $f in A$ such that $M_f$ is flat over $A$) works for finitely presented modules because any such module must "descend" to a finitely presented module over a noetherian (e.g. finitely generated) subring.
              $endgroup$
              – Akhil Mathew
              Jan 15 '11 at 18:41








            4




            4




            $begingroup$
            Thank you for the reference. For the intrested reader, I should mention that Bourbaki proves the following much stronger result: If $M$ is a finitely presented module and if for some prime $p$, the module $M_p$ is free, then there exists a neighborhood $D(f)$ of $p$ such that $M_f$ is free over $A_f$, so that "being free at a point" implies "being free at a neigborhood of a point".
            $endgroup$
            – the L
            Jan 8 '11 at 16:51




            $begingroup$
            Thank you for the reference. For the intrested reader, I should mention that Bourbaki proves the following much stronger result: If $M$ is a finitely presented module and if for some prime $p$, the module $M_p$ is free, then there exists a neighborhood $D(f)$ of $p$ such that $M_f$ is free over $A_f$, so that "being free at a point" implies "being free at a neigborhood of a point".
            $endgroup$
            – the L
            Jan 8 '11 at 16:51




            4




            4




            $begingroup$
            @anonymous: Dear anonymous, more generally, if $M,N$ are finitely presented modules and $M_p, N_p$ are isomorphic, then $M_f simeq N_f$ for some $f in A-p$. This is explained in more detail (and generality) in EGA IV-8, where it is proved that (in a certain sense) if ${R_alpha}$ is an inductive system of rings, then the category of f.p. modules over $varinjlim R_alpha$ is the "colimit of the categories of modules over $R_alpha$," which has many useful applications in reducing questions about arbitrary f.p. modules to questions about finitely generated modules over noetherian rings.
            $endgroup$
            – Akhil Mathew
            Jan 15 '11 at 18:40




            $begingroup$
            @anonymous: Dear anonymous, more generally, if $M,N$ are finitely presented modules and $M_p, N_p$ are isomorphic, then $M_f simeq N_f$ for some $f in A-p$. This is explained in more detail (and generality) in EGA IV-8, where it is proved that (in a certain sense) if ${R_alpha}$ is an inductive system of rings, then the category of f.p. modules over $varinjlim R_alpha$ is the "colimit of the categories of modules over $R_alpha$," which has many useful applications in reducing questions about arbitrary f.p. modules to questions about finitely generated modules over noetherian rings.
            $endgroup$
            – Akhil Mathew
            Jan 15 '11 at 18:40




            2




            2




            $begingroup$
            (contd.) For instance, Grothendieck's generic flatness lemma (if $A$ noetherian integral domain, $B$ finite $A$-algebra, $M$ finite $B$-module, then there is $f in A$ such that $M_f$ is flat over $A$) works for finitely presented modules because any such module must "descend" to a finitely presented module over a noetherian (e.g. finitely generated) subring.
            $endgroup$
            – Akhil Mathew
            Jan 15 '11 at 18:41




            $begingroup$
            (contd.) For instance, Grothendieck's generic flatness lemma (if $A$ noetherian integral domain, $B$ finite $A$-algebra, $M$ finite $B$-module, then there is $f in A$ such that $M_f$ is flat over $A$) works for finitely presented modules because any such module must "descend" to a finitely presented module over a noetherian (e.g. finitely generated) subring.
            $endgroup$
            – Akhil Mathew
            Jan 15 '11 at 18:41











            4












            $begingroup$

            For future reference, I have written up a constructive and reasonably self-contained, if somewhat dense, proof (one page). The basic idea is to first verify that idempotent matrices over local rings are equivalent to diagonal matrices with entries $1$ and $0$, thus showing that finitely generated projective modules over local rings are free.






            share|cite|improve this answer











            $endgroup$













            • $begingroup$
              Well, I think there are simpler proofs for finitely generated projective modules over local rings are free. Never mind, +1.
              $endgroup$
              – user26857
              May 8 '15 at 16:40












            • $begingroup$
              @user26857: I would be interested to hear them! Are they constructive in the technical sense that they avoid the use of the law of excluded middle? I only know of my write-up and the one in Mulvey's gem Intuitionistic algebra and representations of rings.
              $endgroup$
              – Ingo Blechschmidt
              May 8 '15 at 16:54










            • $begingroup$
              math.stackexchange.com/questions/147754/…
              $endgroup$
              – user26857
              May 8 '15 at 17:15










            • $begingroup$
              @user26857: Thanks! The problem is that a proof beginning with "Consider a minimal system of generators" is not constructive, since constructively one cannot show that any inhabited subset of the natural numbers contains a minimal element. In some cases though, the problem can be avoided by a clever rewording; I didn't try that in this case hard enough yet.
              $endgroup$
              – Ingo Blechschmidt
              May 8 '15 at 18:03
















            4












            $begingroup$

            For future reference, I have written up a constructive and reasonably self-contained, if somewhat dense, proof (one page). The basic idea is to first verify that idempotent matrices over local rings are equivalent to diagonal matrices with entries $1$ and $0$, thus showing that finitely generated projective modules over local rings are free.






            share|cite|improve this answer











            $endgroup$













            • $begingroup$
              Well, I think there are simpler proofs for finitely generated projective modules over local rings are free. Never mind, +1.
              $endgroup$
              – user26857
              May 8 '15 at 16:40












            • $begingroup$
              @user26857: I would be interested to hear them! Are they constructive in the technical sense that they avoid the use of the law of excluded middle? I only know of my write-up and the one in Mulvey's gem Intuitionistic algebra and representations of rings.
              $endgroup$
              – Ingo Blechschmidt
              May 8 '15 at 16:54










            • $begingroup$
              math.stackexchange.com/questions/147754/…
              $endgroup$
              – user26857
              May 8 '15 at 17:15










            • $begingroup$
              @user26857: Thanks! The problem is that a proof beginning with "Consider a minimal system of generators" is not constructive, since constructively one cannot show that any inhabited subset of the natural numbers contains a minimal element. In some cases though, the problem can be avoided by a clever rewording; I didn't try that in this case hard enough yet.
              $endgroup$
              – Ingo Blechschmidt
              May 8 '15 at 18:03














            4












            4








            4





            $begingroup$

            For future reference, I have written up a constructive and reasonably self-contained, if somewhat dense, proof (one page). The basic idea is to first verify that idempotent matrices over local rings are equivalent to diagonal matrices with entries $1$ and $0$, thus showing that finitely generated projective modules over local rings are free.






            share|cite|improve this answer











            $endgroup$



            For future reference, I have written up a constructive and reasonably self-contained, if somewhat dense, proof (one page). The basic idea is to first verify that idempotent matrices over local rings are equivalent to diagonal matrices with entries $1$ and $0$, thus showing that finitely generated projective modules over local rings are free.







            share|cite|improve this answer














            share|cite|improve this answer



            share|cite|improve this answer








            edited May 8 '15 at 15:35

























            answered May 8 '15 at 15:01









            Ingo BlechschmidtIngo Blechschmidt

            1,385815




            1,385815












            • $begingroup$
              Well, I think there are simpler proofs for finitely generated projective modules over local rings are free. Never mind, +1.
              $endgroup$
              – user26857
              May 8 '15 at 16:40












            • $begingroup$
              @user26857: I would be interested to hear them! Are they constructive in the technical sense that they avoid the use of the law of excluded middle? I only know of my write-up and the one in Mulvey's gem Intuitionistic algebra and representations of rings.
              $endgroup$
              – Ingo Blechschmidt
              May 8 '15 at 16:54










            • $begingroup$
              math.stackexchange.com/questions/147754/…
              $endgroup$
              – user26857
              May 8 '15 at 17:15










            • $begingroup$
              @user26857: Thanks! The problem is that a proof beginning with "Consider a minimal system of generators" is not constructive, since constructively one cannot show that any inhabited subset of the natural numbers contains a minimal element. In some cases though, the problem can be avoided by a clever rewording; I didn't try that in this case hard enough yet.
              $endgroup$
              – Ingo Blechschmidt
              May 8 '15 at 18:03


















            • $begingroup$
              Well, I think there are simpler proofs for finitely generated projective modules over local rings are free. Never mind, +1.
              $endgroup$
              – user26857
              May 8 '15 at 16:40












            • $begingroup$
              @user26857: I would be interested to hear them! Are they constructive in the technical sense that they avoid the use of the law of excluded middle? I only know of my write-up and the one in Mulvey's gem Intuitionistic algebra and representations of rings.
              $endgroup$
              – Ingo Blechschmidt
              May 8 '15 at 16:54










            • $begingroup$
              math.stackexchange.com/questions/147754/…
              $endgroup$
              – user26857
              May 8 '15 at 17:15










            • $begingroup$
              @user26857: Thanks! The problem is that a proof beginning with "Consider a minimal system of generators" is not constructive, since constructively one cannot show that any inhabited subset of the natural numbers contains a minimal element. In some cases though, the problem can be avoided by a clever rewording; I didn't try that in this case hard enough yet.
              $endgroup$
              – Ingo Blechschmidt
              May 8 '15 at 18:03
















            $begingroup$
            Well, I think there are simpler proofs for finitely generated projective modules over local rings are free. Never mind, +1.
            $endgroup$
            – user26857
            May 8 '15 at 16:40






            $begingroup$
            Well, I think there are simpler proofs for finitely generated projective modules over local rings are free. Never mind, +1.
            $endgroup$
            – user26857
            May 8 '15 at 16:40














            $begingroup$
            @user26857: I would be interested to hear them! Are they constructive in the technical sense that they avoid the use of the law of excluded middle? I only know of my write-up and the one in Mulvey's gem Intuitionistic algebra and representations of rings.
            $endgroup$
            – Ingo Blechschmidt
            May 8 '15 at 16:54




            $begingroup$
            @user26857: I would be interested to hear them! Are they constructive in the technical sense that they avoid the use of the law of excluded middle? I only know of my write-up and the one in Mulvey's gem Intuitionistic algebra and representations of rings.
            $endgroup$
            – Ingo Blechschmidt
            May 8 '15 at 16:54












            $begingroup$
            math.stackexchange.com/questions/147754/…
            $endgroup$
            – user26857
            May 8 '15 at 17:15




            $begingroup$
            math.stackexchange.com/questions/147754/…
            $endgroup$
            – user26857
            May 8 '15 at 17:15












            $begingroup$
            @user26857: Thanks! The problem is that a proof beginning with "Consider a minimal system of generators" is not constructive, since constructively one cannot show that any inhabited subset of the natural numbers contains a minimal element. In some cases though, the problem can be avoided by a clever rewording; I didn't try that in this case hard enough yet.
            $endgroup$
            – Ingo Blechschmidt
            May 8 '15 at 18:03




            $begingroup$
            @user26857: Thanks! The problem is that a proof beginning with "Consider a minimal system of generators" is not constructive, since constructively one cannot show that any inhabited subset of the natural numbers contains a minimal element. In some cases though, the problem can be avoided by a clever rewording; I didn't try that in this case hard enough yet.
            $endgroup$
            – Ingo Blechschmidt
            May 8 '15 at 18:03











            0












            $begingroup$

            This answer shows that if its stalk $M_mathfrak{p}$ at $mathfrak{p}$ is free, then there is an open neighbourhood $mathfrak{p}in D(f)$ on which its value $M_f$ is also free.



            Proof: Reduce to the case that the natural map $Mto M_mathfrak{p}$ is an injection by localising some $f_0in A$. Now use the basis of $M_mathfrak{p}$ to give a surjection $alpha:A^nto M$ whose localisation at $mathfrak{p}$ is the isomorphism $A^n_mathfrak{p}to M_mathfrak{p}$. Thus $ker alpha_mathfrak{p}=0$, so $ker alpha_f=0$ for some $fin A$. Thus $M_fsimeq A^n_f$ is free, completing the proof.



            Thus the quasicompact space $text{Spec}A$ admits a finite cover by $D(f_i)$'s, on each of which $M_{f_i}$ is free. $text{Spec}A=cup D(f_i)$ means precisely that $(f_1,...,f_n)=A$.






            share|cite|improve this answer











            $endgroup$


















              0












              $begingroup$

              This answer shows that if its stalk $M_mathfrak{p}$ at $mathfrak{p}$ is free, then there is an open neighbourhood $mathfrak{p}in D(f)$ on which its value $M_f$ is also free.



              Proof: Reduce to the case that the natural map $Mto M_mathfrak{p}$ is an injection by localising some $f_0in A$. Now use the basis of $M_mathfrak{p}$ to give a surjection $alpha:A^nto M$ whose localisation at $mathfrak{p}$ is the isomorphism $A^n_mathfrak{p}to M_mathfrak{p}$. Thus $ker alpha_mathfrak{p}=0$, so $ker alpha_f=0$ for some $fin A$. Thus $M_fsimeq A^n_f$ is free, completing the proof.



              Thus the quasicompact space $text{Spec}A$ admits a finite cover by $D(f_i)$'s, on each of which $M_{f_i}$ is free. $text{Spec}A=cup D(f_i)$ means precisely that $(f_1,...,f_n)=A$.






              share|cite|improve this answer











              $endgroup$
















                0












                0








                0





                $begingroup$

                This answer shows that if its stalk $M_mathfrak{p}$ at $mathfrak{p}$ is free, then there is an open neighbourhood $mathfrak{p}in D(f)$ on which its value $M_f$ is also free.



                Proof: Reduce to the case that the natural map $Mto M_mathfrak{p}$ is an injection by localising some $f_0in A$. Now use the basis of $M_mathfrak{p}$ to give a surjection $alpha:A^nto M$ whose localisation at $mathfrak{p}$ is the isomorphism $A^n_mathfrak{p}to M_mathfrak{p}$. Thus $ker alpha_mathfrak{p}=0$, so $ker alpha_f=0$ for some $fin A$. Thus $M_fsimeq A^n_f$ is free, completing the proof.



                Thus the quasicompact space $text{Spec}A$ admits a finite cover by $D(f_i)$'s, on each of which $M_{f_i}$ is free. $text{Spec}A=cup D(f_i)$ means precisely that $(f_1,...,f_n)=A$.






                share|cite|improve this answer











                $endgroup$



                This answer shows that if its stalk $M_mathfrak{p}$ at $mathfrak{p}$ is free, then there is an open neighbourhood $mathfrak{p}in D(f)$ on which its value $M_f$ is also free.



                Proof: Reduce to the case that the natural map $Mto M_mathfrak{p}$ is an injection by localising some $f_0in A$. Now use the basis of $M_mathfrak{p}$ to give a surjection $alpha:A^nto M$ whose localisation at $mathfrak{p}$ is the isomorphism $A^n_mathfrak{p}to M_mathfrak{p}$. Thus $ker alpha_mathfrak{p}=0$, so $ker alpha_f=0$ for some $fin A$. Thus $M_fsimeq A^n_f$ is free, completing the proof.



                Thus the quasicompact space $text{Spec}A$ admits a finite cover by $D(f_i)$'s, on each of which $M_{f_i}$ is free. $text{Spec}A=cup D(f_i)$ means precisely that $(f_1,...,f_n)=A$.







                share|cite|improve this answer














                share|cite|improve this answer



                share|cite|improve this answer








                edited Dec 24 '18 at 17:01

























                answered Dec 24 '18 at 16:55









                MeowMeow

                600314




                600314






























                    draft saved

                    draft discarded




















































                    Thanks for contributing an answer to Mathematics Stack Exchange!


                    • Please be sure to answer the question. Provide details and share your research!

                    But avoid



                    • Asking for help, clarification, or responding to other answers.

                    • Making statements based on opinion; back them up with references or personal experience.


                    Use MathJax to format equations. MathJax reference.


                    To learn more, see our tips on writing great answers.




                    draft saved


                    draft discarded














                    StackExchange.ready(
                    function () {
                    StackExchange.openid.initPostLogin('.new-post-login', 'https%3a%2f%2fmath.stackexchange.com%2fquestions%2f16814%2ffinitely-generated-projective-modules-are-locally-free%23new-answer', 'question_page');
                    }
                    );

                    Post as a guest















                    Required, but never shown





















































                    Required, but never shown














                    Required, but never shown












                    Required, but never shown







                    Required, but never shown

































                    Required, but never shown














                    Required, but never shown












                    Required, but never shown







                    Required, but never shown







                    Popular posts from this blog

                    Quarter-circle Tiles

                    build a pushdown automaton that recognizes the reverse language of a given pushdown automaton?

                    Mont Emei